Differentialgleichung

Neue Frage »

tigerteufel Auf diesen Beitrag antworten »
Differentialgleichung
Brauche Hilfe bei der folgenden Aufgabe


Ein Motorboot bewegt sich bei ruhigem Wasser mit einer gleichförmigen Geschwindigkeit von 10 km/h. Bei dieser Geschwindigkeit wird der Motor abgeschaltet und innerhalb von 20 Sekunden verringert sich die Geschwindigkeit des Bootes auf 6 km/h. Es wird angenommen, dass der Widerstand des Wassers das Boot proportional zu seiner Geschwindigkeit bremst. Berechnen sie die Geschwindigkeit des Bootes 2 Minuten nach dem Ausschalten des Motors. Welchen Weg legt das boot in der ersten Minute nach dem Ausschalten des Motors zurück.


Das ist eine differentialgleichung, habe aber nicht mal ne Ahnung wie ich aus dem Text ne Gleichung bekomme. Kann mir dabei jemand helfen?
sqrt(2) Auf diesen Beitrag antworten »

Wichtig ist der Zusammenhang mit Weg , Geschwindigkeit und Beschleunigung . Aufgrunddessen, dass sich die Beschleunigung proportional zur Geschwindigkeit verhält, ist



ein guter Anfang.
tigerteufel Auf diesen Beitrag antworten »

und dann weiter?

Ich habe keinen blassen Schimmer von Physik und sowas.
sqrt(2) Auf diesen Beitrag antworten »

Ich bin nicht hier, um deine Hausaufgaben für dich zu machen... Und dann weiter löst du die Differentialgleichung, was sonst?
tigerteufel Auf diesen Beitrag antworten »

ich will auch nicht, das du die Aufgabe für mich löst, ich hab nur überhaupt keine Ahnung wie ich da rangehen soll.

Diese Differentialgleichungen sind ziemlich verwirrend für mich.

Muss man nicht auch noch den Widerstand berücksichtigen?
sqrt(2) Auf diesen Beitrag antworten »

Du musst schon darüber nachdenken, was mein Lösungsansatz eigentlich bedeutet. Wenn du keine Ahnung von "Physik und sowas" hast, dann musst du die eben bekommen, und das funktioniert nur, wenn du dich mit den angebotenen Lösungen auch auseinandersetzst.

Stell dir einfach mal vor was "Beschleunigung proportional zur Geschwindigkeit" heißt, wenn die Geschwindigkeit abnimmt. Womit ist damit Rechnung getragen, womit nicht?
 
 
tigerteufel Auf diesen Beitrag antworten »

wenn ich jetzt v'=k*v löse komme ich auf v= e^(1/2*k^2) ist das soweit korrekt?

kann ich das dann in die Gleichung v=a*t+v0 einsetzen und k ausrechnen?

da komme ich dann auf die Gleichung: e^(1/2*k^2) =k*e^(1/2*k^2) *t+v0 nach Umformungen komme ich dann auf 1/2*k^2= ln(v0)-ln(1-t+k).
sqrt(2) Auf diesen Beitrag antworten »

Zitat:
Original von tigerteufel
wenn ich jetzt v'=k*v löse komme ich auf v= e^(1/2*k^2) ist das soweit korrekt?

Nein.



Rechts wird nach , nicht nach integriert!

Zitat:
Original von tigerteufel
kann ich das dann in die Gleichung v=a*t+v0 einsetzen und k ausrechnen?

Nein. Wenn du die Differentialgleichung gelöst hast, hast du dein . Deine Gleichung gilt nur für die gleichmäßig beschleunigte Bewegung ( konstant) und würde dir das Lösen einer DGL ersparen.
tigerteufel Auf diesen Beitrag antworten »

wenn ich dv/v=k dt integriere komme ich auf ln(v)=k*t nach v umgestellt ergibt sich v=e^(k*t)

stimmt das jetzt?

aber wie komme ich jetzt auf das was in der Aufgabe gefragt ist, ich habe doch gar kein k gegeben?
sqrt(2) Auf diesen Beitrag antworten »

Zitat:
Original von tigerteufel
wenn ich dv/v=k dt integriere komme ich auf ln(v)=k*t nach v umgestellt ergibt sich v=e^(k*t)

stimmt das jetzt?

Fast, denn du hast beim Integrieren die additive Konstante vergessen.

Zitat:
Original von tigerteufel
aber wie komme ich jetzt auf das was in der Aufgabe gefragt ist, ich habe doch gar kein k gegeben?

Du hast (mit der additiven Konstante) zwei Unbekannte, aber auch zwei Informationen, die sich verwerten lassen (v zum Zeitpunkt t=0s und zum Zeitpunkt t=20s). Zwei Unbekannte, zwei Gleichungen, ist doch toll.
tigerteufel Auf diesen Beitrag antworten »

wie muss ich die Geschwindigkeit vo km/h in m/s umrechen mit 3,6 multiplizieren oder dividieren?

v=e^(k*t)+c

wenn ich dividiere, komme ich am Ende auf c=16/9 und 20*k=ln(-1/9) das ist aber nicht lösbar



mit multiplizieren komme ich auf c=36 und 20*k=ln(-14,4) das ist auch nicht lösbar.


kann das sein?
sqrt(2) Auf diesen Beitrag antworten »

Zitat:
Original von tigerteufel
wie muss ich die Geschwindigkeit vo km/h in m/s umrechen mit 3,6 multiplizieren oder dividieren?

Du kannst sie erst einmal bei km/h lassen. Erst wenn es um die Strecke geht, werden die Einheiten wichtig. (Division wäre übrigens angebracht.)

Zitat:
Original von tigerteufel
v=e^(k*t)+c

Das ist falsch.
tigerteufel Auf diesen Beitrag antworten »

stimmt es muss heißen v=c*e^(k*t) da ich ja ln(v)=k*t+c habe e^c setze ich dann wieder c
sqrt(2) Auf diesen Beitrag antworten »

Exakt.
tigerteufel Auf diesen Beitrag antworten »

jetzt komme ich auf c=10 und k=-0,02554 und was bringt mir das?
sqrt(2) Auf diesen Beitrag antworten »

ist richtig, bei hast du was vergessen.
tigerteufel Auf diesen Beitrag antworten »

heißt das ich habe jetzt für v=e^(-0,02554*t) *10 ? und brauche nur noch die 2 Minuten einsetzen und habe dann die richtige Geschwindigkeit?
tigerteufel Auf diesen Beitrag antworten »

muss ich c nochmal zurück rechnenß oder was meinst du
tigerteufel Auf diesen Beitrag antworten »

ist c=2,3 ?
sqrt(2) Auf diesen Beitrag antworten »

Entschuldige, ich habe mich vertan (ich hatte das an anderer Stelle). ist so richtig.

Zitat:
Original von tigerteufel
heißt das ich habe jetzt für v=e^(-0,02554*t) *10 ? und brauche nur noch die 2 Minuten einsetzen und habe dann die richtige Geschwindigkeit?

Ja.
tigerteufel Auf diesen Beitrag antworten »

das ergibt dann nach 2 Minuten eine Geschwindigkeit von 9,5 km/h wenn ich das einsetze, aber nach 20 sekunden waren es doch nur noch 6 km/h, wie kann das denn dann wieder steigen? jetz bin ich komplett verwirrt
tigerteufel Auf diesen Beitrag antworten »

ich musste die 2 Minuten in Sekunden umrechen. dann kommt eine Geschwindigkeit von v= 0,47 km/h raus.

stimmt das jetztß


und was muss ich machen um den Weg zu bekommen?
sqrt(2) Auf diesen Beitrag antworten »

Die Geschwindigkeit ist so richtig. Was den Weg angeht, habe ich dir ja schon gesagt, dass

.
tigerteufel Auf diesen Beitrag antworten »

Ist das dann ds/dt=v ? damit komme ich dann auf s=v*t+c
sqrt(2) Auf diesen Beitrag antworten »

Das wäre nur richtig, wenn konstant wäre. So kommst du auf

.

Du interessierst dich für die Strecke .
tigerteufel Auf diesen Beitrag antworten »

kann ich dann v=e^(-0,02554*t) *10 einsetzen?

dann ist s(t)=-0,2554 * e^(-0,02554*t) 1 Minute einsetzen fertig?
tigerteufel Auf diesen Beitrag antworten »

ich blicke da nicht durch. am Anfang hast du doch gesagt s''=v'=a und wieso ist jetzt s'=v ?
sqrt(2) Auf diesen Beitrag antworten »

Zitat:
Original von tigerteufel
kann ich dann v=e^(-0,02554*t) *10 einsetzen?

Bitte, bitte, tu mir einen Gefallen und versuche wirklich zu verstehen, was du hier gerade machst, anstatt nur unreflektiert mit irgendwelchen Symbolen herumzuhantieren...

Zitat:
Original von tigerteufel
dann ist s(t)=-0,2554 * e^(-0,02554*t) 1 Minute einsetzen fertig?

Du solltest vorher dein in die richtige Einheit bringen, dann sparst du dir einige Scherereien. Außerdem hast du abgeleitet und nicht integriert.

Zitat:
Original von tigerteufel
ich blicke da nicht durch. am Anfang hast du doch gesagt s''=v'=a und wieso ist jetzt s'=v ?

Wo ist da der Unterschied?
tigerteufel Auf diesen Beitrag antworten »

ich habe jetzt v(t) in m/s umgerechnet v(t)=e^(-0,02554*t) * 25/9


für s(t) ergibt sich dann s(t)=(e^(-0,02554*t) *25/9) / (-0,02554)


s(60)=-23,5 m


warum ist das jetzt negativ? reicht das wenn ich einfach den Betrag nehme?
sqrt(2) Auf diesen Beitrag antworten »

ist nicht, was du suchst, und nein, du kannst nicht den Betrag nehmen.
tigerteufel Auf diesen Beitrag antworten »

dann weiß ich auch nicht weiter. es war doch der weg der in einer Minute zurückgelegt wurde gesucht der irre ich mich da?
sqrt(2) Auf diesen Beitrag antworten »

Denk an die additive Konstante, die hast du bei s(60) zu viel, dein Ergebnis ist also völlig nichtssagend (was man auch daran merkt, dass es negativ ist). Du kennst das doch von der normalen Integration: Obere Grenze minus untere Grenze!
tigerteufel Auf diesen Beitrag antworten »

aber es sind doch gar keine Grenzen angegeben. was soll ich jetzt für v(t) nehmen wenn die additive Konstante zuviel ist?
sqrt(2) Auf diesen Beitrag antworten »

  1. Es sind Grenzen gegeben.
  2. v(t) ist richtig. s(t) fehlt eine additive Konstante.
tigerteufel Auf diesen Beitrag antworten »

welches v war jetzt richtig das erste oder das zweite

v= e^(-0,02554*t) *10 oder v=e^(-0,02554*t)*25/9 und was sind die Grenzen?
tigerteufel Auf diesen Beitrag antworten »

ich glaub jetzt hab ichs


s(t)= integral (e^(-0,02554*t)*25/9) dt in den Grenzen von 0 bis 60 also von 0 bis 1 Minute

s=-23,5-(-108,76) also s=85,26 stimmt's jetzt



oder s(t) = integral (e^(-0,02554*t)*10) dt in den grenzen von 0 bis 60

s=-84,58-(-391,54) also s=306,96 m


ich tendiere eher zum zweiten bin mir aber nicht sicher

oder ist keines davon richtig?
sqrt(2) Auf diesen Beitrag antworten »

Eines ist in einer sinnvollen, das andere in einer wenig sinnvollen Einheit. Finde es selbst heraus.
tigerteufel Auf diesen Beitrag antworten »

wenn ich das erst ergebnis mit 3,6 multipliziere kommt das zweite raus

deshalb denke ich das zweite ist richtig also s=306,9 m
sqrt(2) Auf diesen Beitrag antworten »

Wenn ich 6 mit 9 multipliziere, kommt 54 raus. 54 ist deshalb das richtige Ergebnis.
tigerteufel Auf diesen Beitrag antworten »

Also sind 306 m richtig oder?
Neue Frage »
Antworten »



Verwandte Themen

Die Beliebtesten »
Die Größten »
Die Neuesten »